5
$\begingroup$

Let $L/K$ be a finite Galois extension of number fields that is ramified exactly at one finite prime and is unramified at all infinite primes. Let $U_K$ and $U_L$ denote the units of the ring of integers of $K$ and $L$, respectively. Some examples show that the norm map on units $Norm_{L^*/K^*}: U_L \rightarrow U_K$ is surjective, or equivalently zeroth Tate cohomology of the group $G$ with coefficients in $U_L$ is $0$. Does this surjectivity of the norm map remain true in general for such extensions?

$\endgroup$

1 Answer 1

9
$\begingroup$

A theorem due to Arnold Scholz says that if $K$ has odd class number and $L/K$ is a quadratic extension with a single ramified prime, then the norm map on units is onto.

In general, this does not hold. Take $K = {\mathbb Q}(\sqrt{-5})$ and $L = K(\sqrt{11})$. Then $L/K$ is unramified away from $11$, the unit group of $L$ is $\langle -1, 10 + 3\sqrt{11}\rangle$, and $-1$ is a norm of an element in $L^\times$ by Hasse's norm theorem, but not the norm of a unit.

$\endgroup$
3
  • $\begingroup$ Thanks a lot. What about for the special case $L=\mathbb{Q}(\zeta_{p^n})$ and $K=\mathbb{Q}(\zeta_p)$? I know that in this case the norm map is surjective for regular primes. I expect that it be true also for irregular primes, but I do not know how to prove it. In fact, in this example we have $\hat{H^0}=ker(\epsilon)$ where $ker(\epsilon)$ is the capitulation kernel. Now, regularity of $p$ easily implis that capitulation kernel is $0$. But this method does not work for irregular primes. Note that in this example (which is my goal), surjectivity is equivalent to zeroness of capitulation kernel $\endgroup$ Feb 26 at 15:08
  • $\begingroup$ The statement for regular primes is the exact analog of Scholz's theorem in the quadratic case. I'd start looking for a counterexample rather than for a proof. $\endgroup$ Feb 27 at 14:42
  • $\begingroup$ By a theorem of Iwasawa about cohomology of units of $\mathbb{Z}_p$ -extensions and using direct limit of BRZ exact sequence in my paper on \textit{Ostrowski Quotients for ...}, I am sure that the direct limit over all positive integers $i$ of $ker(\epsilon_i)$ is a finite group for $K=\mathbb{Q}(\zeta_p)$ and $L_i=\mathbb{Q}(\zeta_{p^i})$ (for both regular and irregular primes). Also, I knew the zeroness for regular primes, hence I guessed for irregular primes each $ker(\epsilon_i)$ is zero. $\endgroup$ Feb 28 at 16:45

Your Answer

By clicking “Post Your Answer”, you agree to our terms of service and acknowledge that you have read and understand our privacy policy and code of conduct.

Not the answer you're looking for? Browse other questions tagged or ask your own question.